解析力学/ラグランジアン のバックアップ(No.9)

更新


目次

ラグランジアンの定義

力学系の運動エネルギーを $T$、ポテンシャルエネルギーを $U$ とするとき、 ラグランジアン $L$ を、

$$ L=T-U $$

として定義する。

例えば互いに重力を及ぼし合う2つの星、質量 $M$ の星の座標を $X,Y,Z$、質量 $m$ の星の座標を $x,y,z$、万有引力定数を $G$ とすれば、

$$ T=\frac12 M\bigg[\bigg(\frac{dX}{dt}\bigg)^2+\bigg(\frac{dY}{dt}\bigg)^2+\bigg(\frac{dZ}{dt}\bigg)^2\bigg]+ \frac12 m\bigg[\bigg(\frac{dx}{dt}\bigg)^2+\bigg(\frac{dy}{dt}\bigg)^2+\bigg(\frac{dz}{dt}\bigg)^2\bigg] $$

$$ U=-G\frac{Mm}{\sqrt{(X-x)^2+(Y-y)^2+(Z-z)^2}} $$

なので、

$$ \begin{aligned} L=&\frac12 M\bigg[\bigg(\frac{dX}{dt}\bigg)^2+\bigg(\frac{dY}{dt}\bigg)^2+\bigg(\frac{dZ}{dt}\bigg)^2\bigg]+\frac12 m\bigg[\bigg(\frac{dx}{dt}\bigg)^2+\bigg(\frac{dy}{dt}\bigg)^2+\bigg(\frac{dz}{dt}\bigg)^2\bigg]\rule[-6mm]{0mm}{0mm}\\ &-\bigg[-G\frac{Mm}{\sqrt{(X-x)^2+(Y-y)^2+(Z-z)^2}}\bigg] \end{aligned} $$

である。

力学系の座標と速度

以下では力学系を記述するための座標を $q_1,q_2,\dots,q_n$ で表すことにする。

3次元中での質点1個の運動なら $n=3$ で、$q_1=x,q_2=y,q_3=z$ と考えればいい。

3次元中での質点2個の運動なら $n=6$ で、$q_1=x_1,q_2=y_1,q_3=z_1,q_4=x_2,q_5=y_2,q_6=z_2$ と考えればいい。

質点1が原点から距離 $r$ の球面上を動くなら、球座標を使って $n=2$ として $q_1=\theta,q_2=\phi$ と取れば良い。

また、それらの座標の時間微分を $\dot q_k=\frac{dq_k}{dt}$、 $\ddot q_k=\frac{d^2q_k}{dt^2}$ のように変数上に点を付けて表すことにする。

また、さしあたり運動エネルギーは $q_1,q_2,\dots,q_n$ および $\dot q_1,\dot q_2,\dots,\dot q_n$ の関数、ポテンシャルエネルギーは $q_1,q_2,\dots,q_n$ と $t$ の関数であるとする。すると、

$$ L(q_1,q_2,\dots,q_n,\dot q_1,\dot q_2,\dots,\dot q_n,t)=T(q_1,q_2,\dots,q_n,\dot q_1,\dot q_2,\dots,\dot q_n)-U(q_1,q_2,\dots,q_n,t) $$

の形になる。

カーテシアン座標系($x,y,z$座標系)を思い浮かべていると運動エネルギーに位置座標 $q_1,q_2,\dots,q_n$ が含まれるのはおかしいと感じるかもしれないが、例えば極座標で表すと運動エネルギーに $r$ や $\sin\theta$ が現れる。

ポテンシャルに $t$ を含んでもよいとしているのは、強制振動のように系外から力がかかる場合を表せるようにするためである。

下でも書くように、ラグランジアンにはもっと柔軟な定義もあって、その方がより多くの力学系を記述できるのだが、ここでは話を簡単にするためこの狭義の定義で話を進める。特に断らない限りこの形でない場合でも通じる話になっている。

最小作用の原理

時刻 $t_1$ の座標値 $q_k(t_1)$ と、時刻 $t_2$ の座標値 $q_k(t_2)$ が与えられたとすると、 その間での系の運動 $q_k(t)$ は「作用」と呼ばれる値(ラグランジアンの積分値として与えられる)

$$ S=\int_{t_1}^{t_2}L\ dt $$

を最小にするようなものとなる。というのが最小作用の原理である。

何を言っているかというと、各座標の最初と最後の値が決まってもその間で座標が時間と共にどのように変化するかはいろんな可能性が考えられる。しかし、そのうち実際に起きる運動は $S$ を最小とするものになる、ということだ。(一般に運動方程式は時間に対する2階の微分方程式であるから、1つの座標につき2つの自由パラメータを持つが、始点と終点という2つのパラメータを指定すると運動は1つに定まる(初期座標と初期速度を与えるのと同じ))

え、そんな話は聞いたことがないんだけど、と思うだろうけど実際こういう原理が存在するのだから仕方がないと諦めよう。上記の話がこれまで学んできたニュートン方程式が成り立つ話と矛盾しないことを以下で見る。

ラグランジュの運動方程式

「作用を最小化する」という条件を微分方程式の形に直したものをラグランジュの運動方程式と呼ぶ。

汎関数

$S$ は関数 $q_k(t)$ を1つ決めると1つ値が決まるという意味で「汎関数」と呼ばれる。$S$ が $q_k(t)$ の汎関数であることを強調する際には

$$S=S\big[q_k(t)\big]$$

などと書く。

ある座標の軌跡 $q_k(t)$ を変化させて $q_k(t)+\Delta q_k(t)$ としたとする。$q_k(t)$ が最小作用を与えるという意味は、どのように $\Delta q_k(t)$ を選んだとしても $S$ は元の値より増加する、ということと同義である。

$$S[q_k(t)+\Delta q_k(t)]>S[q_k(t)]$$

ただし始点と終点は決まっているので $\Delta q_k(t_1)=\Delta q_k(t_2)=0$ とする。

$\Delta q_k(t)$ が小さいときを扱うことにして、$S$ を $q_k(t)$ で「微分」することを考える。

汎関数微分

$S$ の $q_k$ による「汎関数微分 $\frac{\delta S}{\delta q_k}$」は、

$$ \begin{aligned} \Delta S&=S\big[q_k(t)+\Delta q_k(t)\big]-S\big[q_k(t)\big]\\ &\sim\int_{t_1}^{t_2}\frac{\delta S}{\delta q_k}\Delta q_k(t)\,dt \end{aligned} $$

として定義される。

普通の関数 $f(x)$ の微分が

$$ \begin{aligned} \Delta f&=f(x+\Delta x)-f(x)\\ &\sim\frac{df}{dx}\Delta x \end{aligned} $$

であるのと比べると、汎関数微分の定義になぜ積分が現れるのか疑問に思うかもしれない。

その疑問は、多変数関数 $f(x_1,x_2,\dots,x_n)$ の偏微分が、

$$ \begin{aligned} \Delta f&=f(x_1+\Delta x_1,\ x_2+\Delta x_2,\ \dots\ ,x_n+\Delta x_n)-f(x_1,x_2,\dots,x_n)\\ &\sim\sum_{k=1}^n\frac{\partial f}{\partial x_k}\Delta x_k \end{aligned} $$

であることと比べると解決するだろう。汎関数 $S$ は $t_1$ から $t_2$ までのすべての時刻に対する $q_k(t)$ の値を引数とする超多変数関数であると見做せるため(個数の引数は連続無限)、それら個々の値の変化に、その値に対する微分値 $\frac{\delta S}{\delta q_k}$ を掛けて足したものが全体としての変化となるわけだ。

ということで、当然だが汎関数微分 $\frac{\delta S}{\delta q_k}$ は時刻の関数となる。

最小点においては汎関数微分がゼロとなる

$f(x)$ の極大点・極小点において $\frac{df}{dx}=0$ が成り立つのと同様に、最小作用を与える $q_k(t)$ に対して、

$$\frac{\delta S}{\delta q_k}=0$$

が成り立つ必要がある。

実際に汎関数微分を計算して $q_k$ に対する条件を求めるにあたっては、$q_k(t)$ を変化させると $\dot q_k(t)$ も変化することを加味する必要がある。さらに $\Delta q_k(t_1)=\Delta q_k(t_2)=0$ を用いた部分積分を適用すると、

$$\begin{aligned} \Delta S &=\int_{t_1}^{t_2}\bigg[\frac{\partial L(\dots,q_k(t),\dot q_k(t),\dots)}{\partial q_k(t)}\Delta q_k(t)+\frac{\partial L(\dots,q_k(t),\dot q_k(t),\dots)}{\partial \dot q_k(t)}\Delta \dot q_k(t)\bigg]\,dt\\ &=\cancel{\bigg[\frac{\partial L(\dots,q_k(t),\dot q_k(t),\dots)}{\partial \dot q_k(t)}\Delta q_k(t)\bigg]_{t_1}^{t_2}}+\\ &\phantom{=}\int_{t_1}^{t_2}\bigg[\frac{\partial L(\dots,q_k(t),\dot q_k(t),\dots)}{\partial q_k(t)}\Delta q_k(t)-\frac{d}{dt}\bigg\{\frac{\partial L(\dots,q_k(t),\dot q_k(t),\dots)}{\partial \dot q_k(t)}\bigg\}\Delta q_k(t)\bigg]\,dt\\ &=\int_{t_1}^{t_2}\underbrace{\bigg[\frac{\partial L(\dots,q_k(t),\dot q_k(t),\dots)}{\partial q_k(t)}-\frac{d}{dt}\bigg\{\frac{\partial L(\dots,q_k(t),\dot q_k(t),\dots)}{\partial \dot q_k(t)}\bigg\}\bigg]}_{=\,\frac{\delta S}{\delta q_k}}\Delta q_k(t)\,dt=0\\ \end{aligned}$$

を得る。これが任意の $\Delta q_k(t)$ に対して成り立つためには汎関数微分 $\frac{\delta S}{\delta q_k}$ をゼロと置く必要があり、次の方程式が得られる。

$$ \frac{d}{dt}\bigg\{\frac{\partial L}{\partial \dot q_k}\bigg\}-\frac{\partial L}{\partial q_k}=0 $$

この方程式はラグランジュの運動方程式と呼ばれ、任意の $t$ および $k$ に対してこの式が成り立つことが「停留作用」の法則の微分形を与える。この方程式に現れる $\frac{\partial L}{\partial \dot q_k}$ や $\frac{\partial L}{\partial q_k}$ は、$\dot q_k$ と $q_k$ とを独立の変数とみなして偏微分したものであることに注意せよ。

ラグランジュの運動方程式自体は作用が最小となることを保証しない。作用が停留値を取ることのみを与える。ラグランジュの運動方程式を解いて得られた軌道の近傍で小刻みに振動するような軌道を考えれば、ポテンシャルエネルギーをほとんど変えないまま運動エネルギーを増加させることがいつでも可能である。したがって、求まる停留値が極大値である可能性はない。ただしそれが鞍点ではないのか、あるいはもし極小値であったとしてもそれが真の最小値であるのか、についてはここまでの議論からは判然としない。

ニュートン方程式が導かれる

ポテンシャルエネルギー $U(x,y,z,t)$ 中で運動する質量 $m$ を持つ質点の運動は、ニュートン方程式

$$ \begin{cases} m\ddot x=-\frac{\partial}{\partial x} U\\ m\ddot y=-\frac{\partial}{\partial y} U\\ m\ddot z=-\frac{\partial}{\partial z} U\\ \end{cases} $$

で記述される。(力は $\bm f=-\bm \nabla U$ で与えられることを思い出せ)

一方、この系の運動エネルギーは $T=\frac{1}{2}m(\dot x^2+\dot y^2+\dot z^2)$ であるから、

$$ L=T-U=\frac{1}{2}m(\dot x^2+\dot y^2+\dot z^2)-U(x,y,z) $$

である。

ラグランジュの運動方程式は、

$$ \begin{cases} \displaystyle\frac{d}{dt}\bigg\{\frac{\partial L}{\partial \dot x}\bigg\}=\frac{\partial L}{\partial x} \ \ \to\ \ \frac{d}{dt}(m\dot x)=-\frac{\partial}{\partial x}U\\ \displaystyle\frac{d}{dt}\bigg\{\frac{\partial L}{\partial \dot y}\bigg\}=\frac{\partial L}{\partial y} \ \ \to\ \ \frac{d}{dt}(m\dot y)=-\frac{\partial}{\partial y}U\\ \displaystyle\frac{d}{dt}\bigg\{\frac{\partial L}{\partial \dot z}\bigg\}=\frac{\partial L}{\partial z} \ \ \to\ \ \frac{d}{dt}(m\dot z)=-\frac{\partial}{\partial z}U \end{cases} $$

となって、確かにニュートン方程式と一致する。

実はラグランジアンや作用を適切に定義することにより、 ニュートン方程式だけでなくマクスウェル方程式や一般相対論などについても 最小作用の形で定式化できることが知られている。

とはいえ例えばローレンツ力をラグランジアンで取り扱うには上記の形では不十分で、$U$ は速度 $\dot q_k$ にも依存する形になる。また、$T-U$ の形ではないラグランジアンを作って摩擦力を表現するようなこともできる。

一般化運動量(正準運動量・力学的運動量)

上で見たとおりラグランジュの運動方程式がニュートン方程式を与える際には

$$ \frac{d}{dt}\bigg\{\underbrace{\frac{\partial L}{\partial \dot q_k}}_\text{運動量}\bigg\}=\underbrace{\frac{\partial L}{\partial q_k}}_\text{力} $$

の形になり、力により運動量が時間変化する、という式になった。

そこで一般の座標 $q_k$ に対しても、

$$ p_k=\frac{\partial L}{\partial \dot q_k} $$

を $q_k$ に対応する運動量であると呼ぶことにする。例えば $q_k$ が極座標の $\theta$ であれば、この値はそもそも通常の運動量と同じ次元すらもたないのだが、それでもこれを $p_\theta$ と書いて運動量として扱う。 そこでこのような「運動量」は通常の運動量と区別するために一般化運動量あるいは正準運動量や力学的運動量と呼ばれる。

このときラグランジュの運動方程式は

$$ \dot p_k=\frac{\partial L}{\partial q_k} $$

を表すことになる。

また、ラグランジアンの全微分は

$$ \begin{aligned} dL &=\sum_{k=1}^n\left[\frac{\partial L}{\partial \dot q_k}d\dot q_k+\frac{\partial L}{\partial q_k}d q_k\right]\\ &=\sum_{k=1}^n\big(p_k\,d\dot q_k+\dot p_k\,d q_k\big)\\ \end{aligned} $$

のように非常に対称性の良い形に表せることになる。

例題

ラグランジュ方程式がニュートン方程式を与えるだけであるなら、なぜ新たにラグランジュ方程式を考える必要があるのだろう。その一端を見るために2つ例題を見てみよう。

$x$ 軸上の点電荷

$x$ 軸上に限り摩擦なく自由に動ける点電荷 $q$ (質量 $m$) が $(0,1)$ に固定された点電荷 $-q$ からのクーロン力と、運動を $x$ 軸上に限るための拘束力のみを受けて運動するとして、点電荷の $x$ 座標の時間変化を表す運動方程式を求めたい。ただし系の誘電率を $\epsilon_0$ とする。

これまでと同様に普通にニュートン方程式を立てるなら、

電荷間の引力は $-\frac{1}{4\pi\epsilon_0}\frac{e^2}{x^2+1}$

その $x$ 軸に沿った成分は $-\frac{1}{4\pi\epsilon_0}\frac{e^2}{x^2+1}\frac{x}{\sqrt{x^2+1}}$

したがって、次の運動方程式を得る。

$$ m\ddot x=-\frac{1}{4\pi\epsilon_0}\frac{e^2}{x^2+1}\frac{x}{\sqrt{x^2+1}} $$

一方、ラグランジアンを使うと、

運動エネルギー $T=\frac{1}{2}m\dot x^2$

ポテンシャルエネルギー $U=\frac{1}{4\pi\epsilon_0}\frac{e^2}{\sqrt{x^2+1}}$

ラグランジアン $L=T-U$

$$ L=\frac{1}{2}m\dot x^2-\frac{1}{4\pi\epsilon_0}\frac{e^2}{\sqrt{x^2+1}} $$

ラグランジュの運動方程式

$$ \frac{d}{dt}\left(\frac{\partial L}{\partial \dot x}\right)-\frac{\partial L}{\partial x}=0 $$

$$ m\ddot x=-\frac{e^2}{4\pi\epsilon_0}\frac{x}{(x^2+1)^{3/2}} $$

として同じ式が得られる。

$x$ に対応する運動量は、

$$ p_x=\frac{\partial L}{\partial \dot x}=m\dot x $$

となって、ちゃんと我々のなじみのある形が出てきた。

振り子の運動

長さ $r$、重りの質量 $m$ の振り子について、その振れ角を $\theta$ として、$\theta$ に対する運動方程式を求めたい。

これまで通りにやるなら重力加速度を $g$ として、

重力 $mg$

その円弧に沿った成分 $-mg\sin\theta$

円弧に沿った加速度 $r\ddot\theta$

より、運動方程式は

$$ mr\ddot\theta=-mg\sin\theta $$

すなわち、次の運動方程式が得られる。

$$ \ddot\theta=-\frac gr\sin\theta $$

一方、ラグランジアンを用いる場合には、

重りの速度 $r\dot\theta$

高さは $r-r\cos\theta$ と表せるから、

運動エネルギー $T=\frac12 mr^2\dot\theta^2$

ポテンシャルエネルギー $U=mgr(1-\cos\theta)$

ラグランジアン $L=\frac12 mr^2\dot\theta^2-mgr(1-\cos\theta)$

$\theta$ に対する運動方程式は、

$$ \frac{d}{dt}\left\{\frac{\partial L}{\partial\dot\theta}\right\}=\frac{\partial L}{\partial\theta} $$

$$ mr^2\ddot\theta=-mgr\sin\theta $$

すなわち、

$$ \ddot\theta=-\frac{g}{r}\sin\theta $$

を得る。

$\theta$ に対応する一般化運動量は、

$$ p_\theta=\frac{\partial L}{\partial \dot \theta}=mr^2\dot\theta $$

となる。

運動をラグランジアン形式で記述することのメリット

ラグランジアンを用いた方法では力の働く方向などを考える必要がなく、 ラグランジアンさえ求めてしまえばそれを形式的に微分するのみで 運動方程式が得られる。

また、運動を記述するのが $x,y,z$ 座標であろうが、$r,\theta,\phi$ であろうが、 基本方程式を同じ形($\dot p_k=\partial L/\partial q_k$)で記述できることも非常に重要である。 この性質を指して、運動方程式が座標変換に対して共変である、という。

ラグランジュの運動方程式の共変性

ラグランジュの運動方程式の共変性は、その元となった最小作用の法則が座標の取り方に寄らない形式になっているため改めて確かめるまでもなく当然なりたつのであるが、ここでは練習も兼ねて明示的に示しておく。

座標系 $q_1,q_2,\dots,q_n$ から別の座標系 $Q_1,Q_2,\dots,Q_n$ への変数変換が、

$$ Q_i=Q_i(q_1,q_2,\dots,q_n)\ \ \ \ \ (i=1,2,\dots,n) $$

で与えられるとする。またこの変換には逆変換が存在して、

$$ q_i=q_i(Q_1,Q_2,\dots,Q_n)\ \ \ \ \ (i=1,2,\dots,n) $$

とする。

この座標変換は $x,y,z$ から $r,\theta,\phi$ のような幾何学的な座標変換であっても良いし、 2つの粒子の座標を重心座標と相対座標とに分けたり、あるいは バネで連結された多数の質点の個々の位置座標から基準振動の振幅と位相へ変換するなど、どんな変換でも構わない。

座標 $q_1,q_2,\dots,q_n$ を決めれば座標 $Q_1,Q_2,\dots,Q_n$ を求められ、 逆に座標 $Q_1,Q_2,\dots,Q_n$ を決めれば座標 $q_1,q_2,\dots,q_n$ を求められる、 ということだけが座標変換の条件である。

このとき、すべての $i=1,2,\dots,n$ に対して、

$$ \frac{d}{dt}\left\{\frac{\partial L}{\partial \dot q_i}\right\}-\frac{\partial L}{\partial q_i}=0 $$

が成り立つことを仮定すると、すべての $i=1,2,\dots,n$ に対して、

$$ \frac{d}{dt}\left\{\frac{\partial L}{\partial \dot Q_i}\right\}-\frac{\partial L}{\partial Q_i}=0 $$

が成り立つことが以下のように導ける。(この話は $q_i$ と $Q_i$ に対して対称なので、当然逆も成り立つ)

まず、$L$ の $Q_i,\dot Q_i$ に対する偏微分を求めよう。

$$ \frac{\partial L}{\partial Q_i} =\sum_{j=1}^n\frac{\partial q_j}{\partial Q_i}\frac{\partial L}{\partial q_j}+\sum_{j=1}^n\frac{\partial\dot q_j}{\partial Q_i}\frac{\partial L}{\partial\dot q_j} $$

$$ \begin{aligned} \frac{\partial L}{\partial\dot Q_i} &=\sum_{j=1}^n\cancel\frac{\partial q_j}{\partial\dot Q_i}\frac{\partial L}{\partial q_j}+\sum_{j=1}^n\frac{\partial\dot q_j}{\partial\dot Q_i}\frac{\partial L}{\partial\dot q_j}\\ &=\sum_{j=1}^n\frac{\partial q_j}{\partial Q_i}\frac{\partial L}{\partial\dot q_j}\\ \end{aligned} $$

第2式の変形は以下の理由で行える。

ここでの偏微分は $\dot Q_j$ と $Q_j$ とを独立の変数と考えて取ることになるので、$q_i$ は $Q_j$ の関数ではあっても $\dot Q_j$ の関数ではなく、$\frac{\partial q_j}{\partial\dot Q_i}=0$ である。また、

$$ \dot q_i=\sum_{j=1}^n \frac{\partial q_i}{\partial Q_j}\dot Q_j $$

より、

$$ \frac{\partial\dot q_i}{\partial\dot Q_j}=\frac{\partial q_i}{\partial Q_j} $$

であるのだ。

上式を代入すると、

$$ \begin{aligned} \frac{d}{dt}\left\{\frac{\partial L}{\partial \dot Q_i}\right\}-\frac{\partial L}{\partial Q_i} &=\sum_{j=1}^n\left[\frac{d}{dt}\left\{\frac{\partial q_j}{\partial Q_i}\frac{\partial L}{\partial \dot q_j}\right\}-\frac{\partial q_j}{\partial Q_i}\frac{\partial L}{\partial q_j}-\frac{\partial\dot q_j}{\partial Q_i}\frac{\partial L}{\partial\dot q_j}\right]\\ &=\sum_{j=1}^n\bigg[\frac{d}{dt}\left\{\frac{\partial q_j}{\partial Q_i}\right\}\frac{\partial L}{\partial \dot q_j}+\underbrace{\frac{\partial q_j}{\partial Q_i}\frac{d}{dt}\left\{\frac{\partial L}{\partial \dot q_j}\right\}-\frac{\partial q_j}{\partial Q_i}\frac{\partial L}{\partial q_j}}_{q_j,\dot q_j\text{に対する運動方程式よりゼロ}}-\frac{\partial\dot q_j}{\partial Q_i}\frac{\partial L}{\partial\dot q_j}\bigg]\\ &=\sum_{j=1}^n\bigg[\frac{\partial }{\partial Q_i}\frac{d}{dt}q_j-\frac{\partial\dot q_j}{\partial Q_i}\bigg]\frac{\partial L}{\partial \dot q_j}=0\\ \end{aligned} $$

最下行冒頭の等号では $Q_i$ による偏微分と時間による全微分を入れ替えた。

ハミルトニアンの定義とエネルギーの保存

任意の $L$ に対して

$$ \begin{aligned} \delta L=\frac{dL}{dt}\delta t=& L(q_1(t+\delta t),q_2(t+\delta t),\dots,q_n(t+\delta t),q_1(t+\delta t),q_2(t+\delta t),\dots,q_n(t+\delta t),t+\delta t)\\ &-L(q_1(t),q_2(t),\dots,q_n(t),\dot q_1(t),\dot q_2(t),\dots,\dot q_n(t),t)\\ =&\sum_{i=1}^n\bigg[\frac{\partial L}{\partial q_i}\dot q_i+\frac{\partial L}{\partial\dot q_i}\ddot q_i+\frac{\partial L}{\partial t}\bigg]\delta t\\ =&\sum_{i=1}^n\bigg[\frac{d}{dt}\bigg\{\frac{\partial L}{\partial\dot q_i}\bigg\}\dot q_i+\frac{\partial L}{\partial\dot q_i}\ddot q_i+\frac{\partial L}{\partial t}\bigg]\delta t\\ =&\bigg[\frac{d}{dt}\bigg\{\sum_{i=1}^n\frac{\partial L}{\partial\dot q_i}\dot q_i\bigg\}+\frac{\partial L}{\partial t}\bigg]\delta t\\ \end{aligned} $$

すなわち、

$$ \frac{d}{dt}\bigg\{\bigg(\sum_{i=1}^n\frac{\partial L}{\partial\dot q_i}\dot q_i\bigg)-L\bigg\}=-\frac{\partial L}{\partial t} $$

が成り立つ。そこで $\{\ \ \}$ の中味を

$$ H=\bigg(\sum_{i=1}^n\frac{\partial L}{\partial\dot q_i}\dot q_i\bigg)-L=\bigg(\sum_{i=1}^n p_i\dot q_i\bigg)-L $$

と置いて、この値をハミルトニアンと呼ぶ。すると、

$$ {}-\dot H=\frac{\partial L}{\partial t} $$

と書けるから、もし右辺がゼロ、すなわちラグランジアンが陽に時間に依存していなければ、 ハミルトニアンは時間に対する保存量となる。右辺がゼロでなければこれは単位時間当たりの ハミルトニアンの減少量が $\frac{\partial L}{\partial t}$ で書けることを表す。

さて、この保存量は物理的には何に当たるのだろうか? 実は $L=T-U$ と書ける場合、ハミルトニアンは系の全エネルギーに相当する。

$L=T-U$ と書ける場合、ラグランジアンが陽に時間に依存するというのはポテンシャル $U$ が時間に依存する場合であり、この時、系外にポテンシャルを変化させている「誰か」が存在して、その誰かと系との間でエネルギーのやりとりが行われている場合に相当する。

逆にポテンシャルが時間に依存しないことは系を孤立系と見做せることと同義であり、そのとき系のエネルギーが保存する、というのが上記の結果である。

ハミルトニアンが確かに系の全エネルギーに相当することを上の例で試しておこう。

$x$ 軸上の点電荷の運動

$$ \begin{aligned} H&=m\dot x\times \dot x-(\frac12m\dot x^2-U)\\ &=\frac12m\dot x^2+U\\ &=T+U \end{aligned} $$

振り子の運動

$$ \begin{aligned} H&=mr^2\dot \theta\times \dot \theta-(\frac12mr^2\dot \theta^2-U)\\ &=\frac12mr^2\dot \theta^2+U\\ &=T+U\\ \end{aligned} $$

ポテンシャルとエネルギー

$L=T-U$ と書ける場合、上のようにして求めたハミルトニアンは、

$$ H=\bigg(\sum_{i=1}^n p_i\dot q_i\bigg)-L=\bigg(\sum_{i=1}^n p_i\dot q_i\bigg)-T+U $$

となるから、$\displaystyle\bigg(\sum_{i=1}^n p_i\dot q_i\bigg)=2T$ であれば

$$ H=T+U $$

となり、期待通りハミルトニアンが運動エネルギーとポテンシャルエネルギーの和として表せる。

実際、ポテンシャルエネルギー $U$ が $\dot q_k$ を含まず、また運動エネルギー $T$ が $\dot q_k^2$ の線形結合で

$$ T=\sum_{i=1}^n \frac12m_i(q_1,q_2,\dots,q_n)\dot q_i^2 $$

の形に表されるとき(ここでの $m_i$ は一般に座標の関数であり質量そのものに限定しない。上の振り子の例の $T=\frac12mr^2\dot\theta^2$ であれば $\frac12mr^2$ を指す)、上記の項は、

$$ \begin{aligned} \sum_{i=1}^n p_i\dot q_i&=\sum_{i=1}^n \frac{\partial L}{\partial \dot q_i}\dot q_i\\ &=\sum_{i=1}^n \frac{\partial T}{\partial \dot q_i}\dot q_i\\ &=\sum_{i=1}^n m_i\dot q_i^2=2T\\ \end{aligned} $$

となり、運動エネルギーの2倍 $2T$ に等しくなる。

カーテシアン座標系における質点の運動エネルギーはまさに上記の形で $m_i$ を質点の質量としたものに等しいし、剛体系で物体の回転を表す回転角を座標に取った場合にも、その回転エネルギーは上記の形となる。

さらに、後に見るように$\sum_{i=1}^n p_i\dot q_i$の値は座標系の取り方によらない、すなわち同じ系を異なる座標 $P_i,Q_i$ で表した場合にも $\sum_{i=1}^n P_i\dot Q_i=\sum_{i=1}^n p_i\dot q_i=2T$ となるから、もっと変った座標を取ったとしても、この値が運動エネルギーの2倍に等しいことは変らない。

一方で、ローレンツ力を含む場合のように $U$ が速度 $\dot q_i$ を含む場合などはそのようにうまく行かず、 「ハミルトニアンは $T+U$ と等しくならない」。

それでも $L=T-U$ と書けるときには $H$ は系の全エネルギーに等しいため*1たぶん成り立つのだと思うのだけれど、ちゃんと証明するにはどうしたらいいのか、個人的に理解できてない、 そのようなケースでラグランジアンに現れる「ポテンシャル」$U$ は、全エネルギーを計算するためには使えない、あくまで力を計算するために定義された便宜的なものとなる。ローレンツ力を扱う際の $U$ がその代表である。

まとめると、

  • $L=T-U$ が成り立ち、$\displaystyle\bigg(\sum_{i=1}^n p_i\dot q_i\bigg)=2T$ であれば系のエネルギーは $H=T+U$ と表せる。
  • $L=T-U$ が成り立つものの、$U$ が速度を含むような場合には $\displaystyle\bigg(\sum_{i=1}^n p_i\dot q_i\bigg)\ne 2T$ となり、系のエネルギーは $H\ne T+U$ である。すなわちこの場合の $U$ は本来の意味でのポテンシャルではない。

これらのケースでは、

$$ {}-\dot H=\frac{\partial L}{\partial t} $$

は時間当たりに系から失われるエネルギーが $\frac{\partial L}{\partial t}$ で与えられることを表している。

一方、摩擦力を無理矢理ラグランジアンで表すような場合などでは、そもそも $L=T-U$ の形ではないラグランジアンを取る。そのような場合ハミルトニアンは系のエネルギーとは異なる量になるのであるが、その場合にも、

$$ {}-\dot H=\frac{\partial L}{\partial t} $$

は系のエネルギー変化と関係が深いものになる。

質問・コメント





*1 たぶん成り立つのだと思うのだけれど、ちゃんと証明するにはどうしたらいいのか、個人的に理解できてない

Counter: 44499 (from 2010/06/03), today: 6, yesterday: 0